Multivalued!

Algebra Level 4

Given real numbers a , b , c a, b, c constrained by a 2 + b 2 + c 2 = 1 a^2+b^2+c^2=1 , the minimum of a b + b c + c a ab+bc+ca is attained for

Four sets of values of a , b , c a, b, c Two sets of values of a , b , c a, b, c Three sets of values of a , b , c a, b, c More than four sets of values of a , b , c a, b, c

This section requires Javascript.
You are seeing this because something didn't load right. We suggest you, (a) try refreshing the page, (b) enabling javascript if it is disabled on your browser and, finally, (c) loading the non-javascript version of this page . We're sorry about the hassle.

2 solutions

Atomsky Jahid
Jul 6, 2020

a 2 + b 2 + c 2 + 2 ( a b + b c + c a ) = ( a + b + c ) 2 a^2+b^2+c^2 +2(ab+bc+ca) = (a+b+c)^2 a b + b c + c a = 1 2 ( a 2 + b 2 + c 2 ) + 1 2 ( a + b + c ) 2 ab+bc+ca = - \frac{1}{2} (a^2+b^2+c^2) + \frac{1}{2} (a+b+c)^2 a b + b c + c a = 1 2 + 1 2 ( a + b + c ) 2 ab+bc+ca = - \frac{1}{2} + \frac{1}{2} (a+b+c)^2

Therefore, to minimize a b + b c + c a ab+bc+ca , we need to minimize ( a + b + c ) 2 (a+b+c)^2 . ( a + b + c ) 2 0 (a+b+c)^2 \geq 0 m i n ( ( a + b + c ) 2 ) = 0 \therefore min( (a+b+c)^2 ) = 0 a + b + c = 0 \implies a+b+c = 0

We have the following set of equations now. a + b + c = 0 c = a b a+b+c = 0 \> \implies \> c=-a-b a 2 + b 2 + c 2 = 1 a^2+b^2+c^2 = 1 a 2 + b 2 + ( a b ) 2 = 1 a^2+b^2+(-a-b)^2 = 1 b 2 + a b + a 2 1 2 = 0 b^2 + ab + a^2 - \frac{1}{2} = 0 b = a ± 2 3 a 2 2 b = \frac{ -a \pm \sqrt{2-3a^2} }{2}

Also, c = a b c = -a - b c = a 2 3 a 2 2 c = \frac{ -a \mp \sqrt{2-3a^2} }{2}

Hence, we get the triple as, ( a , a ± 2 3 a 2 2 , a 2 3 a 2 2 ) \Bigg( a, \frac{ -a \pm \sqrt{2-3a^2} }{2}, \frac{ -a \mp \sqrt{2-3a^2} }{2} \Bigg)

As ( a , b , c ) (a, b, c) should be a triple of real numbers, 2 3 a 2 0 \sqrt{2-3a^2} \geq 0 2 3 a 2 3 \implies - \sqrt{\frac{2}{3}} \leq a \leq \sqrt{\frac{2}{3}}

Notice, we solved for the triple ( a , b , c ) (a,b,c) with respect to a a . We could have solved the system with respect to b b or c c too. Hence, from the symmetry argument: 2 3 a , b , c 2 3 - \sqrt{\frac{2}{3}} \leq a,b,c \leq \sqrt{\frac{2}{3}}

Finally, as we can see from the expression of the triple, we can choose any arbitrary a a from the range [ 2 3 , 2 3 ] [ - \sqrt{\frac{2}{3}}, \sqrt{\frac{2}{3}}] , and we would get a triple of real numbers which satisfy our conditions. Therefore, we have infinite sets of values.

Starting from the identity ( a + b + c ) 2 = a 2 + b 2 + c 2 + 2 a b + 2 b c + 2 c a {\left( {a + b + c} \right)^2} = {a^2} + {b^2} + {c^2} + 2ab + 2bc + 2ca , we get ( a + b + c ) 2 = 1 + 2 a b + 2 b c + 2 c a a b + b c + c a = ( a + b + c ) 2 1 2 . {\left( {a + b + c} \right)^2} = 1 + 2ab + 2bc + 2ca \Rightarrow ab + bc + ca = \frac{{{{\left( {a + b + c} \right)}^2} - 1}}{2}. The expression ( a + b + c ) 2 1 2 \frac{{{{\left( {a + b + c} \right)}^2} - 1}}{2} gets minimised when ( a + b + c ) 2 {\left( {a + b + c} \right)^2} does so, that is when a + b + c = 0 a + b + c = 0 .
There are infinitely many triples of real numbers to satisfy this.

In fact, due to the restriction given, a solution is any triple that corresponds to the coordinates of a point that belongs to the circle which is the intersection of the plane z + y + z = 0 z+y+z=0 ant the sphere x 2 + y 2 + z 2 = 1 x^2+y^2+z^2=1 .

The set of all these triples is S = { ( 1 2 cos t + 1 6 sin t , 1 2 cos t + 1 6 sin t , 2 3 sin t ) , t [ 0 , 2 π ) } . S = \left\{ {\left( { - \frac{1}{{\sqrt 2 }}\cos t + \frac{1}{{\sqrt 6 }}\sin t,\;\frac{1}{{\sqrt 2 }}\cos t + \frac{1}{{\sqrt 6 }}\sin t,\; - \sqrt {\frac{2}{3}} \sin t} \right),\quad t \in \left[ {0,\;2\pi } \right)} \right\}.

In a different perspective, the solutions may be written as { ( k , 1 2 ( 2 3 k 2 k ) , 1 2 ( 2 3 k 2 k ) ) , k [ 2 3 , 2 3 ] } . \left\{ {\left( {k,\;\frac{1}{2}\left( {\sqrt {2 - 3{k^2}} - k} \right),\;\frac{1}{2}\left( { - \sqrt {2 - 3{k^2}} - k} \right)} \right),\quad k \in \left[ { - \sqrt {\frac{2}{3}} ,\;\sqrt {\frac{2}{3}} } \right]} \right\}.

For some concrete examples, all 6 permutations of ( 0 , 1 2 , 1 2 ) \left( {0,\;\frac{1}{{\sqrt 2 }},\; - \frac{1}{{\sqrt 2 }}} \right) are solutions.

Hence, the answer is More than four sets of values of a, b, c . \boxed{\text{More than four sets of values of a, b, c}}.

No restriction on the values of a , b a, b and c c ? I mean, can any triple satisfying a + b + c = 0 a+b+c=0 will minimize a b + b c + c a ab+bc+ca ? In which range a , b , c a, b, c should lie?

A Former Brilliant Member - 11 months, 1 week ago

You are right. Not any triple will do. I'll add some more on this in my solution.

Thanos Petropoulos - 11 months, 1 week ago

0 pending reports

×

Problem Loading...

Note Loading...

Set Loading...